Wie erhalte ich die Position zum Zeitpunkt ttt, wenn die Beschleunigung von der Position abhängt? [Duplikat]

Bei meinem Besuch einer realistischeren Partikelbewegungsanimation F = k Q 1 Q 2 D 2 , F = M A , So:

A = k Q 1 Q 2 M D 2
Meine Geschwindigkeit, die das Obige integriert (von einer Website, weil ich vergessen habe, wie es geht), ist v = k Q 1 Q 2 M D + v 0 . Seltsamerweise ist das Integral davon , das eine Verschiebung (!) geben sollte D = D 0 + ( v 0 D ) ( k Q 1 Q 2 / M ) ( l N | D | ) . Funktioniert es? Wenn ja, was muss ich tun, um es zu bekommen v , D oder A zu einer bestimmten Zeit t? (Ich dachte daran, zu bekommen W = F D und der Versuch, Arbeit mit Zeit durch eine andere Gleichung zu verbinden, vielleicht Macht?, etc., aber jetzt F variiert und hängt auch davon ab D (duh...) und ich habe irgendwie aufgehört zu wissen, was ich tun soll. Das meiste (praktisch alles?) Zeug, das ich gesehen habe, befasst sich entweder mit variierender Beschleunigung in Bezug auf die Zeit oder mit Kreisbewegungen. Vielen Dank im Voraus :)

NACHTRAG: Das sieht schwerkraftbezogenen Fragen verdächtig nahe, aber in meinem Fall habe ich Ionen oder geladene Teilchen im Sinn, bei denen sowohl Massen als auch die Bewegungen beider Körper eine Rolle spielen. Was das Thema aufwarf, war die Frage "waren Ionen harte Kugeln, wenn sie 'kollidieren', ist der Abpraller eine Tatsache, oder ist die Anziehungskraft so groß, dass sie bereits ein -v erzeugt, das höher als v aus der Impulserhaltung des Abprallers resultiert die Kollision? Wenn es einen "Rückprall" gibt, wie ist es? usw. ". (Mein ursprüngliches Modell hat nur einen Li+ und einen F- 6000 Picometer voneinander entfernt in einem 1 Kubikmeter großen „Universum“, falls Sie sich fragen, was ich mache.)

Die Gleichung für die Beschleunigung ist eine inhomogene Differentialgleichung zweiter Ordnung und ich weiß ehrlich gesagt nicht, wie ich sie jetzt lösen soll. Ich weiß jedoch, dass das, was Sie hier getan haben, nicht richtig ist; Weil v = D X / D T . Also um zu finden X , müssen Sie integrieren v über T (nicht X oder D , wie Sie es hier getan haben.) dh X = v D T v D D (Dieser Fehler ist beim Finden passiert v auch.) Darüber hinaus ist die Dimension der endgültigen Antwort nicht korrekt.
Obwohl es aus dem Titel wahrscheinlich nicht ersichtlich ist, ist die Frage im Grunde dieselbe: Berechnen Sie die Position als Funktion der Zeit, wenn Sie eine Kraft mit umgekehrtem quadratischem Gesetz haben.
Aha, ich hatte das Gefühl, dass ich aus dieser Sache nicht so einfach herauskommen würde (Übersetzung: hör auf mit dem Programmieren und schlage den Kalkül). Außerdem, ja, ich habe sogar ein bisschen bei der Elektrostatik auf den Gravitationsseiten geschummelt, die meistens g als konstant betrachten, und diejenigen, die dies nicht tun, sind nicht sehr benutzerfreundlich: O Und ich dachte, dass Newton und Coulomb so alt sind, dass ich wollte war mittlerweile trivial. Ich werde mir die hässliche Formel aus dem möglichen Duplikat ansehen. Ich bin ein Programmierer, und die meisten Formeln verlieren sowieso ihren ästhetischen Algebra-Look, sobald sie in Code konserviert sind. Vielen Dank :)

Antworten (1)

A = D v D T

A = D v D T D X D X

A = D v D X D X D T

A = v D v D X

X 0 X A . D X = v 0 v v . D v

Hier, X ist nicht die endgültige Position; es ist eine Variable. Beachten Sie, dass wir die Differentialgleichung für einen allgemeinen Fall lösen.

2 X 0 X A . D X = v 2 v 0 2

v 2 = v 0 2 + 2 X 0 X A . D X

v = v 0 2 + 2 X 0 X A . D X

D X D T = v 0 2 + 2 X 0 X A . D X

0 T D T = X ich X F 1 v 0 2 + 2 ( X 0 X A . D X ) D X

Hier, X F ist die Endstellung. Wenn Sie eine allgemeine Gleichung benötigen, können Sie ersetzen X F mit X aber achten Sie darauf, die obere Grenze des äußeren Integrals nicht mit der oberen Grenze des inneren Integrals zu verwechseln. Das sind zwei verschiedene Dinge und die obere Grenze des inneren Integrals muss um des äußeren Integrals willen eine Variable sein.

Vielen Dank! Meine Mathematik ist Bronzezeit, aber indem ich "sie nicht verwirre", habe ich verstanden, dass Sie das Innere als var (was mir -2kq1q2/x geben würde) und nicht als auszuwertende Obergrenze auswerten wollten, und dann , mit dem zu integrieren Sie die äußere (yikes). Ein Online-Integrator-Spiel mir eine nette Gleichung, bei der v0 nicht 0 sein kann, was mich fragen ließ, ob ich das Richtige getan habe oder ob wir gerade die Eigenschaft "Partikel können nicht ruhen" auf gruselige Weise aufgedeckt haben :) ( Es war schon unheimlich, dass wir keine Zeit brauchen , aber wir müssen es für menschliche Zwecke einfügen und es ist nicht einfach ...)
Was hast du ersetzt A um Partikel zu bekommen, können nicht in Ruhe sein?
Mein Fehler. Es ist so, dass die Software mir ein letztes Integral gegeben hat, bei dem v0^3 am unteren Ende eines Bruchs steht, aber ich sah später, dass das obere Ende auch Null ist, wenn d und v0 Null sind. Ich betrachte dieses Integral nun schon seit einiger Zeit und frage mich: "Wage ich es zu fragen, wie man eine Verschiebung in Bezug auf die Zeit erhält?" Die endgültige Gleichung, die ich erhalten habe und die (kx q1 x q2/Masse)=C, v=Anfangsgeschwindigkeit und d=Abstand ergibt, war: t=[ C* (ln(∣ √(v^2d−2Cd)+v ∣) − ln( ∣ √(v^2d−2Cd)−v ∣)) +vd√(v^2d−2Cd) ] / v^3 (Ich entschuldige mich im Namen des Universums dafür, da ich noch lernen muss Formatierung...).
Um mehr über Mathjax zu erfahren, lesen Sie bitte MathJax Basic Tutorial and Quick Reference .